Oral ENS Ulm, 36 planches - Page 2 — Les-mathematiques.net The most powerful custom community solution in the world

Oral ENS Ulm, 36 planches

245

Réponses

  • @etanche, je connais quelqu'un qui a eu la planche 22 et c'est bien $y'=a+b$. D'ailleurs, il y a une coquille : $a$ est uniformément continue.
  • @calli merci pour ton retour, la personne qui a eu cet exo peut-elle nous donner la solution
    ou des indications ?


    Planche 27 exercice 1

    C'est un Miklos-Schweitzer math competition 1971 problème 6 voir ici

    https://artofproblemsolving.com/community/c7h235963p1298846
  • Une indeed pour le premier exercice de la planche 11 concernant « les triades »?
  • Planche 34 exercice 2

    On peut utiliser la fonction de Lambert

    $x^{f(x)}=f(x)$ a pour solution $f(x)= \frac{-W(-\ln(x))}{\ln(x)}$
  • Planche 30 exercice 1 , première question

    Voir le nouveau livre de Francinou Gianiella Nicolas oraux X-ENS nouvelle édition juin 2019
    page 241


    Planche 31 exercice 2

    C'est le théorème de Nisnevich-Brysgalov 1953

    Voir dans le livre page 62
    Auteur Victor Prasolov
    Titre Problème théorème algèbre linéaire
    traduit par Éric KOURIS
    Éditeur Cassini
  • @etanche : pour la planche 22, deuxième question. Supposons que $y'(t)$ ne tend pas vers $0$, alors $a(t)$ non plus, donc (quitte à changer de signe) il existe $\epsilon>0$ tel que pour tout $t_0>0$ il existe $t>t_0$ tel que $a(t)>4\epsilon$.

    Il existe $t_1>t_0$ tel que pour tout $t> t_1$ on a $|b(t)|<\epsilon$. Il existe $\eta>0$ tel que pour tous $t,t'$ on a $|t-t'|\leqslant \eta\implies |a(t)-a(t')|<\epsilon$. On a alors pour $t>t_1$ tel que $a(t)>4\epsilon$ et tout $t'\in [t,t+\eta]$ : $y'(t')\geqslant \epsilon$ donc $y(t+\eta)-y(t)\geqslant\eta\epsilon$.
  • @etanche : je suis très impressionné par ta rapidité à retrouver des énoncés dans ces annales de compétitions (dont je ne connaissais même pas l'existence). Est-ce que tu as un « truc » ?
  • etanche écrivait : http://www.les-mathematiques.net/phorum/read.php?4,1841908,1842048#msg-1842048
    [Inutile de recopier un message présent sur le forum. Un lien suffit. AD]
    onjour étanche, je ne vois pas où vous voulez en venir.
  • Il est vrai que la rapidité avec laquelle etanche identifie la provenance des exercice est époustouflante. Je croyais naïvement que les examinateurs étaient chargé de créer une trentaine d'exercice chaque année afin de renouveler le stock. En tout cas je reste admiratif de la capacité des examinateurs (de très haut niveau) de proposer des exercices qui ne sont pas des redites d'exercices classiques. Même plusieurs années après les concours la plupart de ces exercices restent difficiles ...
  • Planche 11, exercice 1 :

    Pour les cercles, c'est un problème classique.
    Supposons qu'un tel recouvrement soit possible. Soit $d_1$ le rayon d'un des cercles, disons $C_1$. Il faut recouvrir la zone intérieur à $C_1$. Si l'on considère deux cercles intérieurs à $C_1$, l'un d'eux, disons $C_2$, doit avoir un diamètre $d_2$ inférieur à $\dfrac{d_1}{2}$ puisque tous les cercles sont deux à deux disjoints. On recommence et on construit ainsi une suite de cercles $C_i$ de diamètre $d_i$ tels que $C_{i+1}$ est à l'intérieur de $C_i$ et $d_{i+1} < \dfrac{d_i}{2}$. On construit ainsi une suite décroissante de disques fermés, dont la suite des diamètres tend vers $0$. L'intersection de tous ces disques est donc réduite à un point, qui ne peut appartenir à aucun des cercles. Contradiction.

    Pour les triades, on prouve le résultat de Moore (voir mon message précédent qui contient un lien et une preuve). Il reste à voir qu'une triade est un fermé d'intérieur vide. D'après le théorème de Baire, le plan ne peut donc être la réunion dénombrable de triades.

    Question perso : le théorème de Baire est-il au programme ?

    Pierre.
  • Non, le théorème de Baire n'est pas au programme. Et il ne fait pas non plus partie des résultats hors programme mais proches du programme (au contraire des suites de Cauchy ou des espaces métriques, par exemple).
  • Planche 19 exercice 1 , première question
    Voir le théorème 3.1 et la remarque 2.2 de l'article Dan Shemesh , common eigenvector of two matrices
    dans linear algebra and its application
    Volume 62 , november 1984 , page 11-18
    https://www.sciencedirect.com/science/article/pii/0024379584900855


    Pour la deuxième question de la planche 19 exercice 1
    voir la réponse de Loupblanc ici

    https://math.stackexchange.com/questions/1899110/can-the-same-vector-be-an-eigenvector-of-both-a-and-at
  • Planche 15 exercice 2
    Voir proposition 8 de l'article de Gérald Bourgeois.

    Common invariant subspace of commuting matrices
    Linear algebra and its applications , volume 438 , issue 7 , April 2013 , page 3030-3038



    https://reader.elsevier.com/reader/sd/pii/S0024379512008488?token=4E56C4E1ABECD61E0CD8FAE912B7C37012807CCB17C7DC61430B90C39BD7469DEB14027363DFBE84A6B56AD6100EB7AE


    Pour aller plus loin sur les équations matricielles de la forme XA-AX=f(X)
    voir l'article de

    Gérald Bourgeois, how to solve the matrix equation XA-AX=f(X)
    Linear algebra and its applications , volume 434, 2011, page 657-668

    https://reader.elsevier.com/reader/sd/pii/S0024379510005008?token=B8B1E314C709AE348972F1BCE5AD35C35005FD703884E62F80B30F1F1A3E6A3E05989B4FDDB74442F74F3668E7DDABEE
  • Planche 11 , exercice 2

    Avec la fonction de Lambert on a $x=-nW(-1/n)$
  • @etanche : Planche 6 exercice 2
    Êtes-vous sûr que le groupe de Prüfer est un contre-exemple ? Je pense que vous avez mal lu la question.
  • Il me semble que Gentil a raison pour la planche 6, exercice 2 :

    Soit $G$ un groupe infini ayant un nombre fini de sous-groupes. Par principe des tiroirs, on dispose d'une partie infinie $A$ de $G$ telle que tout élément de $A$ engendre un même sous-groupe $H$ de $G$. Mais alors $H$ est monogène et infini, donc isomorphe à $(\mathbb Z,+)$, donc il admet une infinité de sous-groupes.
  • @simeon et gentil merci pour vôtre vigilance


    Planche 5 exercice1 et 24 vont ensemble

    Notions utilisés les quasi-morphismes

    la suite $f(g^n)/n$ est convergente de limite F(g)

    Dans l'article de Jean Claude Picard voir page 117-118
    cohomologie bornée des surfaces
    Bull.Soc.math.France
    Vol 125 , 1997 , page 115-142


    Koji Fujiwara
    Quasi-homomorphism on mapping class group
    voir page 2 à 4
    Tout quasi-morphisme s'écrit comme une somme d'un quasi-morphisme homogène et d'une fonction bornée.

    à compléter ...
  • Planche 20 exercice 1

    C'est un exercice de AMM American Mathematical Monthly
    le numeros 11976 du volume de janvier 2019 page 89.

    @ Éric si tu passes par là peux-tu nous trouver le corriger et le poster , merci à toi.
  • Voila la solution du problème 11976, publiée dans le Monthly de janvier 2019.
  • Planche 25 exercice 2

    C'était postés sur une autre forum en mars 2019

    Voir la réponse de soccolo pour une solution

    ici

    https://artofproblemsolving.com/community/c7h1802531_polynomial_and_matrix
  • Planche 36, exercice 2 :

    Déjà le la fonction rang est solution, mais aussi toutes les fonctions du type $a \operatorname{Rang} +b$. Mais il me semble que si on se donne une suite croissante $a_0, \dots, a_n$ alors la fonction définie par $f(A)=a_k$ si $\operatorname{A}=a_k$ est solution. Du coup il suffirait de montrer que ce sont les seules.
    Première observation $f(A)\leq f(I_n)$ pour toute matrice et $f(0) \leq f(A)$ pour toute matrice. Ensuite si $P$ est inversible on a $f(P) \leq f(I_n) \leq f(P)$ de sorte que toute matrice inversible $P$ satisfait $f(P)=f(I_n)$. Ensuite si $A$ et $B$ sont de même rang, on écrit $A=PBQ$ et alors $f(A) \leq f(BQ)\leq f(B)$ et finalement $f(B)=f(A)$.
    Avec ça j'espère qu'on a une solution.

    Planche 5, exercice 2 : il me semble que c'est plus ou moins Ascoli
  • @mickael merci pour ta contribution
  • Planche 4 exercice 1

    Si quelques personnes veulent écrire la solution et poster, merci d'avance.

    J'ai l'impression qu'il faut prouver l'inégalité pour k=2
    Puis par recurrence avec l'inégalité bien connue
    $n-rg(A)+rg(AB)\geq rg(B)$ et peut-être q'il faut aussi
    utiliser le rang d'un produit de matrices est plus petit que le rang
    de chaque matrice
    B=I-A(1)...A(k) et A=A(1)
  • La planche 6 exercice 1. m'a l'air "classique" et difficile, quelqu'un aurait une référence ou une idée de preuve ?

    Mise à jour : en fait on montre qu'une telle fonction est analytique en montrant à la main la convergence du reste de Taylor (par la positivité) ensuite on prend $z_0$ un premier zéro et on écrit $f=\sum \frac
    {f^{(n)(z_0)}}{n!}(z-z_0)^n$, si $z_1$ est un autre zéro, par positivité on en déduit que tous les coefs sont nuls et donc $f=0$.

    @etanche : quand comptes-tu utiliser l'hypothèse que les matrices sont des projecteurs ? D'ailleurs plutôt que de récurrer, l'identité que tu donnes pour deux matrices, si on refait sa démonstration pour trois matrices on obtient $r(ABC) \geq r(AB)+r(BC)-rg(B)$ et ça permet de démontrer le cas $k=3$.
    En fait on utilise l'hypothèse de projecteur comme ça je crois : si $A$ est un projecteur alors $r(A)+r(I-A)=n$ car $r(I-A)=dim ker (A)$, mais là on voudrait $r(AB)+r(I-AB) \leq n$, il doit y avoir un truc du genre.
  • @Mickaël, pour la planche 6 exercice 1, on commence par supposer par l'absurde que $f$ a au moins deux zéros. On pose $a = \sup \{ x \in [0,1] \mid f(x) = 0 \}$. Donc $a>0$ et $f_{|[0,a]} = 0$ car $f$ est positive et croissante. D'où : $\forall n, f^{(n)}(a) = 0$. Ainsi, avec la formule de Taylor appliquée en $a$ : $\forall x \in ]a,1[, \forall n, f(x) = \int_a^x \frac{(x-t)^n}{n!} f^{(n+1)}(t) \mathrm{d}t \leqslant \int_a^1 \left(\frac{x-t}{1-t}\right)^n \frac{(1-t)^n}{n!} f^{(n+1)}(t) \mathrm{d}t \leqslant x^n \int_a^1 \frac{(1-t)^n}{n!} f^{(n+1)}(t) \mathrm{d}t = x^n f(1) \xrightarrow[n \rightarrow \infty]{} 0$. Donc $f=0$ par continuité en 1.
  • Je crois que tu as du écrire ta réponse en même temps que j'ai modifié mon message pour répondre à ma question, désolé !

    Pour l'exercice avec les projecteurs, pour $k=2$ et en utilisant l'hypothèse qu'on est en présence de projections, avec $A'=I-A, B'=I-B$, l'énoncé est éguivalent à $r(A')+r(B') \ge r(A'B'-A'-B')$ qui est vérifiée car $r(A'B'-A'-B')=r(A'(B'-I)-B') \le r(A'(B'-I))+r(B') \le r(A')+r(B')$. Normalement l'autre message permet de traiter le cas général.
  • Bonjour,
    Je cherche l'exercice 1 de la planche 22.

    • Pour l'espérance de $\det A$, je trouve $0$ car elle vaut $\displaystyle \sum_{\sigma \in \mathfrak{S}_n} \varepsilon(\sigma)$ à un facteur multiplicatif près ou, plus simplement, car transposer deux colonnes de $A$ ne change pas sa loi.

    • Pour le deuxième moment, j'écris :
    \begin{align*}
    \mathbb{E}\big[(\det A)^2\big] &= \mathbb{E} \,\bigg[\displaystyle \sum_{\sigma \in \mathfrak{S}_n} \varepsilon(\sigma) \prod_{i = 1}^{n} A_{i, \sigma(i)} \, \sum_{\tau \in \mathfrak{S}_n} \varepsilon(\tau) \prod_{j = 1}^{n} A_{j, \tau(j)}\,\bigg] \\

    &= \displaystyle \sum_{\sigma \in \mathfrak{S}_n} \sum_{\tau \in \mathfrak{S}_n} \varepsilon(\sigma \tau) \, \mathbb{E}\,\bigg[\prod_{i = 1}^{n} A_{i, \sigma(i)} \Big( \prod_{j \neq i} A_{j, \tau(j)} \Big) A_{i, \tau(i)}\,\bigg] \\

    &= \displaystyle \sum_{\sigma \in \mathfrak{S}_n} \sum_{\tau \in \mathfrak{S}_n} \varepsilon(\sigma \tau) \mathbb{E}\bigg[\prod_{\sigma(i) \neq \tau(i)}^{n} A_{i, \sigma{i}} A_{i, \tau{i}} \prod_{\sigma(i) = \tau(i)}^{n} A_{i, \sigma(i)}^2 \prod_{j \neq i} A_{j, \tau(j)}\bigg] \hspace{-3cm}\\

    &= \displaystyle \sum_{\sigma \in \mathfrak{S}_n} \sum_{\tau \in \mathfrak{S}_n} \varepsilon(\sigma \tau) \prod_{\sigma(i) \neq \tau(i)} p^2 \prod_{\sigma(i) = \tau(i)} p \prod_{j \neq i} p \\

    &= \displaystyle \sum_{\sigma \in \mathfrak{S}_n} \sum_{\tau \in \mathfrak{S}_n} \varepsilon(\tau) \prod_{\sigma(i) \neq \sigma^{-1} \circ\tau(i)} p^2 \prod_{\sigma(i) = \sigma^{-1} \circ \tau(i)} p \prod_{j \neq i} p\,, &\text{car }\tau \mapsto \sigma^{-1} \tau \text{ est une bijection à }\sigma\text{ fixé} \\

    &= \displaystyle \sum_{\sigma \in \mathfrak{S}_n} \sum_{\tau \in \mathfrak{S}_n} \varepsilon(\tau) \prod_{\sigma(i) \neq \tau(i)} p^2 \prod_{\sigma(i) = \tau(i)} p \prod_{j \neq i} p\, , &\text{car }\sigma \mapsto \sigma^2 \text{ est une bijection} \\

    &= \displaystyle \sum_{\tau \in \mathfrak{S}_n} \varepsilon(\tau) \sum_{\sigma \in \mathfrak{S}_n} p^{2n - \mathrm{card}\{i \in [1, n],\, \sigma(i) = \tau(i)\}} p^{n - 1} \\

    &= p^{3n - 1} \displaystyle \sum_{\tau \in \mathfrak{S}_n} \varepsilon(\tau) \sum_{\sigma \in \mathfrak{S}_n} p^{- \mathrm{card}\{i \in [1, n],\, \sigma(i) = \tau(i)\}}

    \end{align*} J'ignore cependant comment calculer le cardinal apparaissant.
    J'ai essayé de fixer $\tau$ et de compter les $\sigma$ ayant $0$ point en commun avec $\tau$ puis $1$ jusqu'à $n$ mais sans succès.
  • Je pense qu'il suffit de découper ta somme sur les $\sigma$ en parties $P_k$ telles que $|\{i \text{ t.q. } \sigma (i)=\tau(i)\}|=k$ et de dénombrer l'ensemble des $\sigma$ qui satisfait ça.

    Des gens qui font un peu de probas pour le 23 ?
  • Je trouve que pour chaque $k \geq 1$ il y a ${n \choose k} (n - k)! = \frac{n!}{k!}$ permutations ayant $k$ points en commun avec $\tau$ (on choisit les $k$ points parmi $n$ et on multiplie par le nombre de permutations d'un ensemble de $n - k$ éléments) et pour $k = 0$ il y en a $(n - 1)!$ mais l'ennui c'est que ça me donnerait une variance nulle alors que je ne pense pas que le déterminant soit dégénéré
  • Planche 18 exercice 2


    Cet exercice était sur un autre forum , voir la solution de Ghoshadi qui m'a l'air juste( pouvez-vous confirmer)


    https://artofproblemsolving.com/community/c7h1853706_a_limit_containing_improper_integral
  • Sevaus : on avait déjà abordé ce problème il y a quelques mois, au moins pour le cas $p=1/2$.

    L'espérance de $\det (A^2)$ ne peut pas être nulle car $\det(A^2)=\det(A)^2\geq 0 $ et que le déterminant de l'identité vaut $1$.
  • Planche 2 exercice 1, le polynôme donné par etanche est nul et donc en particulier $A^n=0$ et on montre aussi que $B^n=0$. Ça n'a pas l'air suffisant, mais au moins on peut prendre n'importe quel sev, son image par $A$ et par $B$ perdra en dimension. Cela dit il faudrait que l'image d'un tel sous-espace soit incluse dans le même sous-espace ...
    Affaire à suivre pour les amateurs.

    Edit : je dis n'importe quoi.
  • Une approche de la planche 1.

    J'avais fabriqué un problème analogue mais plus simple, avec le nombre de « Pile » égal au nombre de « Face », et j'en avais parlé ici je ne sais plus quand. Il est plus facile et fait intervenir les nombres de Catalan. C'est lié au problème du scrutin, de Joseph Bertrand. On peut aborder un tel problème en représentant une suite de $P, F$ par un trajet sur un quadrillage.

    Sur le dessin, on a calculé de proche en proche le nombre $T(m,n)$ de trajets de l'origine au point de coordonnées $(m,n)$ pour $m \in \mathbb N, n \in \mathbb N$, au cours desquels l'ordonnée n'est jamais égale au double de l'abscisse. C'est le nombre de suites de $P, F$ avec $m$ fois $ F$ et $n$ fois $P$ au cours desquelles on n'a jamais le nombre de $P$ égal au double du nombre de $ F$.

    On obtient ces valeurs par : $T(m,1)=T(1,n)=1$, et $T(m,n)=T(m,n-1)+T(m-1,n)$, sauf $T(m,2m)=0$.
    Par exemple $T(2,3)=4$ correspond aux suites : $PPPFF, PFFPP, FPFPP, FFPPP$.

    Soit $a_n$ le nombre de suites de $P, F$ à $3n$ termes, dans lesquelles le nombre de « Pile » est égal au double du nombre de « Face » pour la première fois au $3n$-ème lancer. On a : $a_n=T(n-1,2n)+T(n,2n-1)$, d'où d'après le dessin : $a_1=1+2=3, a_2=2+4=6, a_3=7+14=21, a_4=30+60=90, a_5=143+286=429$.
    Par exemple $a_2=6$ correspond aux suites : $PPPPFF, PPPFPF, PPPFFP, PFFPPP, FPFPPP, FFPPPP$.

    La probabilité pour que le nombre de « Pile » soit égal au double du nombre de « Face » pour la première fois au $3n$-ème lancer est : $\frac{a_{n}}{2^{3n}}$.
    Et la probabilité que les lancers s'arrêtent est donc : $\displaystyle \overset{+\infty }{\underset{n=1}{\sum }}\frac{a_{n}}{2^{3n}}$.

    À suivre...

    Bonne journée, déjà fin juillet.
    31/07/2019
  • Planche 22 exercice 1
    http://www.les-mathematiques.net/phorum/read.php?12,1800546
    Source indiquée par Corto
  • Pour la planche 1, il y a une solution ici.
  • Suite de mon précédent message. http://www.les-mathematiques.net/phorum/read.php?4,1841908,1843420#msg-1843420
    J'ai interrogé OEIS pour ma suite $a_n$, et il semble que ce soit la suite A024485
    https://oeis.org/A024485
    Son expression est : $ a_{n}=\frac{2}{3n-1}(_{~n}^{3n})$. Des super-Catalan, en quelque sorte.
    Alors pour conclure, il faut calculer la somme de la série entière : $\displaystyle f(x)=\sum^{+\infty }_{n=1} a_{n}x^{n}$. Ceci se fait d'ordinaire en formant une équation différentielle linéaire dont la fonction $f$ est solution, et en résolvant ladite équation. Hugh, beaucoup à faire...

    Autre piste. Le dessin montre que : $T(n,2n-1)=2T(n-1,2n)$. Si l'on arrive à calculer $T(m,n)$, on pourra conclure.
    À suivre...
  • Planche 30 exercice 1 , deuxième question

    Ce type de décomposition est dite la représentation de Lorentz d'un polynôme

    G.G.Lorentz
    Degree approximation by polynomials with positive coefficients
    Math.Ann. Volume 151 , année 1963 page 239-251

    Pouvez-vous trouver l'article ci-dessus en pdf et le poster ? Merci
  • Bonjour,

    Juste pour dire : sur les 36 planches, je suis incapable de faire quoi que ce soit sur disons 30, et à peine capable de rédiger les 6 autres.

    Quel niveau faut-il maîtriser pour savoir rédiger disons 18 planches ? Est-ce L3 ?
  • Bonjour,

    C'est à l'issue d'un parcours un peu sportif que je pense avoir atteint une solution de l'exercice $1$ de la planche $22$ évoqué dans un précédent message.

    $\begin {align*}\mathbb E \left ( \text{Det} A^2\right )& = \displaystyle \sum _{\sigma, \tau \in \mathfrak S_n} \varepsilon (\sigma \tau)\: \mathbb E \left (\prod_{i=1}^n A_{i,\sigma(i)} A_{i,\tau (i)} \right ) = \sum_{\sigma , \tau \in \mathfrak S_n} \varepsilon (\sigma \tau)\prod _{i=1}^n \mathbb E (A_{i, \sigma(i)} A_{i, \tau(i)}) \:\:\quad \text{ (indépendance des }\:A_{i,\sigma(i)}A_{i, \tau(i)}) \\ &= n! \sum_{\sigma \in \mathfrak S_n} \varepsilon (\sigma) p ^{2n -f(\sigma)} \:\:\:\quad \quad \text{où}\: f(\sigma) := \text {Card} \Big\{ i \in [\![1;n]\!] \mid \sigma(i) =i \Big \}\\ & = n!\: p^n\sum_{k=0}^n p^{n-k}(f_k^+ - f_k^-) \quad \text{où}\: f_k^+: =\text{Card} \Big\{ \sigma \in \mathfrak An \mid f(\sigma) = k\Big\},\quad f_k^-: = \text{Card} \Big\{ \sigma \in \mathfrak S_n \setminus \mathfrak A_n \mid f(\sigma) = k \Big \}. \end{align*} $

    $f_k^+ = \binom nk d_{n-k}^+, \quad\: f_k^- = \binom nk d_{n-k}^- \quad$ où $d_k^+\:$ (resp. $d_k^-$) désigne le nombre de "dérangements pairs" (resp. impairs) de $\mathfrak S_k \quad(d_0^+=1, \:d_0^- =0)$.
    $$\mathbb E \left(\text{Det }A^2 \right) =\displaystyle n!\: p^n \sum_{k=0}^n p^k\binom nk (d_k^+ - d_k^-) .\quad \quad (\star)$$

    Or on dispose de la relation: $\:\: \forall n \geqslant 2,\:\: \dfrac {n!}2 = \text{Card} \:\mathfrak A_n = \displaystyle \sum _{k=0}^n \binom nk d_k^+ =\sum _{k=0}^n \binom nk d_k^-$ qui entraine: $\displaystyle \sum _{k=0} ^n \dfrac 1 {(n-k)!} \dfrac{ (d_k^+ - d_k^-)}{k!} =0 ,$
    puis: $\exp( X )\left(\displaystyle \sum_{n=0}^{+\infty} \dfrac {d_n^+-d_n^-}{n!}X^n\right) =1+X, \quad \forall n \in \N, \:\: d_n^+-d_n^- = (-1)^{n-1} (n-1).\quad $ Avec $(\star)$ et $q=1-p,\:$ on parvient alors à:
    $$\boxed { \mathbb E \left ( \text{Det} \:A^2 \right) = n! \:p^n\: q^{n-1} \:(np +q). }$$
  • Pour l'exercice 1 de la planche 2, les matrices $A, B$ vivent dans un espace vectoriel de matrice non inversible donc dans un espace de dimension au plus $n(n-1)$. Avec ça on devrait pouvoir conclure ... je l'espère
  • Toujours concernant la planche 2 exercice 1, voici le début du raisonnement pour ceux que ça intéresse.
    La condition $\det(A+t_iB)=0$ pour $n+1$ points entraîne que $A+tB$ est non inversible pour toute valeur de $t$, cela ayant été évoqué auparavant. Je pense que la bonne manière de voir cela est de dire que $A,B$ appartiennent à un sous-espace vectoriel $V \subset M_n(\mathbb{R})$ formé uniquement de matrices non inversibles. On peut supposer que $A=\mathrm{Diag}(I_{n-1},0) \in V$. Écrivons les matrices $M \in V$ sous forme bloc $M=\begin{bmatrix}A&B \\ C & D\end{bmatrix}$. En considérant $M-tA$ (qui est non inversible) et la fonction $t \mapsto \det(M-tA)=0$ obtient de la nullité des coefs du polynôme que $D=0$ et $CB=0$. Donc concrètement on a $$
    M=\begin{bmatrix}A&(b_i) \\ (c_i) & 0\end{bmatrix},
    $$ avec $\sum b_ic_i=0$. Avec ça on a un peu avancé, mais pas non plus tant que ça en fait. En travaillant un peu on montre que $\dim(V)\leq n(n-1)$, mais comment l'utiliser ? Bien sûr si on imagine que $V$ est formé des matrices de dernière colonne nulle il suffit de prendre $V$ comme la droite engendrée par le dernier vecteur de la base canonique et $W=\{0\}$, mais sinon ça ne me paraît pas clair.

    Mise à jour : je ne sais plus comment on montre exactement que $dim(V) \leqslant n(n-1)$ mais on est plus très loin. Ensuite quitte à enrichir $V$ avec de nouvelles matrices on pourra supposer $dim(V)=n(n-1)$ et alors il me semble avoir lu un résultat il y a un moment comme quoi un tel espace est un idéal maximal de l'algèbre $M_n$ et peut donc se voir comme les matrices avec une dernière colonne nulle, et donc ça nous irait parfaitement. Quelqu'un sait comment démontrer ce que je dis ? Ou bien une autre piste ?
  • @mickael et side merci pour vos solutions pour planche 2 exercice 1
    @simeon merci pour ta contribution planche 1 et 6
    On remercie aussi gentil , bobby joe ,eric, lou16, chaurien, sevaus, corto,PierreB,calli,jpmjpmjpm,
    totem,JLT,poirot, krokop,alea ,jandri pour leur contribution
  • Exerice 31 : (premier exercice)

    - On procède par récurrence sur $m\geq 1.$
    - Pour le cas $m=1,$ on note $a_{1}=e^{i\theta}.$
    Par hypothèse, on a $$\vert e^{i\theta}-1\vert =\vert e^{in\theta}-e^{i(n+1)\theta}\vert \rightarrow_{n\rightarrow +\infty} 0.
    $$ On trouve alors $\theta=0\mbox{ }[2\pi]$ i.e. $a_{1}=1.$
    - Supposons que la suite $\displaystyle \left(A_{n}:=\sum_{k=1}^{m+1}a_{k}^{n}\right)_{n\geq 0}$ (où chacun des $a_{k}\in\mathbb{T}$) converge pour un certain $m\geq 1.$
    * Cette suite converge nécessairement vers $m+1.$ Montrons donc que $m+1$ est une valeur d'adhérence de cette suite.

    Quitte à extraire en cascade et à renuméroter les termes de la suite, il existe une extraction $(\phi(n))$ telle que pour tout $k,$ $(a_{k}^{\phi(n)})$ converge (disons vers $l_{k}\in\mathbb{T}$) et telle que la suite $(\phi(n+1)-\phi(n))$ est strictement croissante.
    Ainsi, $$A_{\phi_{n+1}-\phi(n)}\rightarrow_{n\rightarrow +\infty} \sum_{k=1}^{m+1}l_{k}\overline{l_{k}}=\sum_{k=1}^{m+1}1=(m+1).

    $$ **Prenons une valeur d'adhérence de la suite $(a_{1}^{n})$ que l'on note $l_{1}\in\mathbb{T}.$

    Quitte à extraire en cascade, il existe une extraction $(\phi(n))$ telle que pour tout $k,$ $(a_{k}^{\phi(n)})$ converge (disons vers $l_{k}\in\mathbb{T}$).
    On a alors en passant à la limite : $\displaystyle \sum_{k=1}^{m+1}l_{k}=m+1.$
    Mais alors, par le cas d'égalité de l'inégalité triangulaire $l_{k}=l_{1}$ (comme tous les nombres en jeu sont de module $1$) et donc $l_{1}=1.$

    Ainsi, la suite $(a_{1}^{n})$ est convergente (vers $1$) et le premier pas de la récurrence nous indique que $a_{1}=1.$
    ***On conclut par récurrence car la suite $\displaystyle \left(\sum_{k=2}^{m+1}a_{k}^{n}\right)_{n\geq 0}$ est convergente (vers $m$).

    Ainsi, $\displaystyle \forall k\in \{1,\ldots,m+1\},\mbox{ } a_{k}=1.$
  • Exercice 14 (deuxième exercice)

    Dans les deux preuves, on va procéder par récurrence sur le degré $n\geq 1$ de $P.$

    Le cas $n=1$ est facile car alors $\displaystyle \|P\|_{\infty}:=\sup_{z\in\mathbb{T}}\vert P(z)\vert=1+\vert \alpha_{1} \vert$ et comme $\|P\|_{\infty}\leq 1,$ il vient $\alpha_{1}=0$ et donc $P(z)=z.$

    Première preuve :
    On applique Gauss-Lucas pour avoir que les racines de $P'$ vivent encore dans $\displaystyle \mathbb{D}=\left\{z\in\mathbb{C}\mbox{ }|\mbox{ } \vert z\vert <1\right\}.$
    On écrit alors $\displaystyle P'(z)=n\prod_{k=1}^{n-1}(z-\beta_{k}):=nQ(z)$ où pour tout $k,$ $\beta_{k}\in\mathbb{D}.$

    Mais alors par l'inégalité de Bernstein, on a $\displaystyle \|P'\|_{\infty}\leq n\|P\|_{\infty}\leq n.$
    Ainsi, $\|Q\|_{\infty}\leq 1.$

    Par hypothèse de récurrence, il vient alors $Q(z)=z^{n-1}$ et en intégrant, on a alors $P(z)=z^{n}-\alpha_{1}^{n}$ (par exemple).
    Comme $\|P\|_{\infty}=1+\vert \alpha_{1}\vert ^{n}\leq 1,$ il vient alors $\alpha_{1}=1$ et donc $P(z)=z^{n}.$

    Deuxième preuve : (plus "self-contained")

    *Soit $n\geq 1.$ Premièrement, si $\displaystyle P(z)=\sum_{k=0}^{n}a_{k}z^{k}$ avec $a_{n}\neq 0$ alors $\displaystyle \|P\|_{\infty}\geq \vert a_{0} \vert + \vert a_{n}\vert.$

    (petite remarque au passage, cette inégalité -du moins son cas d'égalité- encode le théorème d'équioscillation des polynômes de Tchebytchev)

    En effet, considérons $\lambda\in\mathbb{T}$ et $\omega=e^{\frac{2i\pi}{n}}.$
    On a alors par un calcul direct : $\displaystyle \sum_{k=1}^{n}P(\lambda\omega^{k})=n(\lambda^{n}a_{n}+a_{0}).$
    En appliquant l'inégalité triangulaire et en choisissant convenablement $\lambda\in\mathbb{T},$ il vient $\displaystyle \|P\|_{\infty}\geq \vert a_{0} \vert + \vert a_{n}\vert.$

    **On a alors l'inégalité suivante : $\displaystyle 1+\prod_{k=1}^{n}\vert \alpha_{k} \vert \leq \|P\|_{\infty} \leq 1.$
    Ainsi, l'une des racines $\alpha_{k}$ du polynôme $P$ est nulle. Quitte à renommer les racines de $P,$ on a alors $\alpha_{1}=0.$ On écrit alors $\displaystyle P(z)=z\prod_{k=2}^{n}(z-\alpha_{k}):=zQ(z).$
    Mais alors, on a $\displaystyle \|Q\|_{\infty}\leq 1.$
    On conclut par récurrence que $Q(z)=z^{n-1}$ et donc $P(z)=z^{n}.$
  • Pour ceux que ça intéresse, puisque ce fil a déjà donné une solution ou un lien vers une solution, les exercices pour lesquels rien n'a été dit sont les suivants :
    3.1
    3.2
    4.1 (En cours)
    8.1
    8.2
    9.1
    10.2
    13.1
    14.1
    16.2
    17.2
    19.2 (manque la fin)
    19.3
    21.1
    21.2
    23
    25.1
    26
    28.1
    32.3
    34.1
    35.2
    36.1

    Il y a dans cette liste encore quelques exercices simples ou classiques.
  • Planche 35 (premier exercice)

    Soit $n\geq 1.$ Fixons $\varepsilon>0.$
    Avec un peu de débombrement puis en appliquant l'inégalité de Bienaîmé-Tchebytchev, on a successivement :
    \begin{align*}
    \displaystyle \mathbb{P}\left(\vert \frac{S_{n}}{n} \vert \geq \varepsilon \mbox{ } |\mbox{ } S_{2n}=0\right) & \displaystyle =\frac{\mathbb{P}\left(\vert \frac{S_{n}}{n} \vert \geq \varepsilon \mbox{ }\cap \mbox{ } S_{2n}=0\right)}{\mathbb{P}(S_{2n}=0)}\\
    & \displaystyle \leq \frac{\mathbb{P}\left(\vert \frac{S_{n}}{n} \vert \geq \varepsilon\right)}{\frac{\binom{2n}{n}}{2^{2n}}}\\
    & \displaystyle \leq \frac{\mbox{Var}(S_{n})}{\varepsilon^{2}n^{2}\frac{\binom{2n}{n}}{2^{2n}}}\\
    & \displaystyle \leq \frac{1}{\varepsilon^{2}n\frac{\binom{2n}{n}}{2^{2n}}}.

    \end{align*} Or, par la formule de Stirling, on a : $$ \frac{\binom{2n}{n}}{2^{2n}}\sim \frac{1}{n\sqrt{\pi}}.

    $$ Il vient alors pour une certaine constante $C_{\varepsilon}>0,$ $$ \mathbb{P}\left(\vert \frac{S_{n}}{n} \vert \geq \varepsilon \mbox{ } |\mbox{ } S_{2n}=0\right)\leq \frac{C_{\varepsilon}}{\sqrt{n}}\rightarrow_{n\rightarrow +\infty} 0.

    $$ Ainsi, par encadrement, la limite recherchée est donc nulle.
  • En réponse à Chaurien à propos de la planche 1:

    pour le cas "égalité des piles et des faces":
    Course à l'égalité

    pour le cas "deux fois plus de piles que de faces":
    Piles double des faces

    et plus récemment:
    Attente:nombre de piles soit double des faces
  • Planche 32 (exercice 1) :

    Enoncé légèrement modifié où la fonction $f$ est supposée seulement $C^{\infty}$ mais la limite simple $g$ plus régulière...

    Je vais utiliser le lemme de Baire (car c'est tout de même plus facile avec! ^^) et aussi supposer que $g$ est $C^{1}$... Tout simplement car je ne sais pas faire (après réfléxion) le cas donné dans l'énoncé (qui a la "saveur" du théorème de Sunyer-Ballagher après tout...)

    *Soit $R>0.$
    Notons pour $n\geq 1,$ $A_{n}=\left\{x \in [-R,R]\mbox{ }|\mbox{ } \forall k\geq 0,\mbox{ } \vert f^{(k)}(x)\vert \leq n \right\}.$

    Par continuité de chacune des applications $f^{(k)},$ $A_{n}$ est donc un fermé de $[-R,R]$
    Comme $\displaystyle \bigcup_{n\geq 1}A_{n}=[-R,R],$ il existe $n_{0}$ tel que $A_{n_{0}}$ soit d'intérieur non vide et même, à nouveau par une application du lemme de Baire, $\mathcal{U}:=\displaystyle \bigcup_{n\geq 1}\mbox{Int}(A_{n})$ est un ouvert dense de $[-R,R].$

    **Soit $x\in\mathcal{U}.$ Il existe alors $r>0$ tel que $I_{x}:=[x-r,x+r]\subset A_{n}$ pour un certain $n\geq 1.$
    Soit $y\in I_{x}.$ On a alors pour tout $k\geq 0,$ $$f^{(k)}(y)-f^{(k)}(x-r)=\int_{x-r}^{y}f^{(k+1)}(t)dt.$$ Par convergence dominée, on obtient alors $\displaystyle g(y)-g(x-r)=\int_{x-r}^{y}g(t)dt.$

    Il vient alors en dérivant la relation précédente : $g'(y)=g(y).$

    ***On a alors que $g'=g$ sur $\mathcal{U}.$ Comme $g$ est $C^{1}$ et que l'ouvert $\mathcal{U}$ est dense dans $[-R,R],$ on obtient $g'=g$ sur $[-R,R].$
    Le raisonnement précédent étant valide pour tout $R>0,$ on a alors $g'=g$ sur $\mathbb{R}.$
    Et ainsi, il existe $\lambda\in \mathbb{C}$ tel que pour tout $x\in\mathbb{R},$ $\displaystyle g(x)=\lambda e^{x}.$
Connectez-vous ou Inscrivez-vous pour répondre.
Success message!